Is the Divergence of the Cross Product of Gradients Zero?

  • Thread starter Thread starter Payne0511
  • Start date Start date
  • Tags Tags
    Identity Vector
Click For Summary
To prove that div(grad f x grad g) equals zero, one can start by expressing the gradients in component form and taking their cross product. The divergence of this cross product can be simplified using the property of mixed partial derivatives being equal. By applying index notation and the Levi-Civita symbol, it is shown that both terms in the resulting expression vanish due to the antisymmetry of the Levi-Civita symbol. Alternatively, known vector identities can be utilized to arrive at the same conclusion. Thus, the divergence of the cross product of gradients is indeed zero.
Payne0511
Messages
5
Reaction score
0

Homework Statement


div(grad f x grad g)=0. I need to prove this somehow.


Homework Equations





The Attempt at a Solution



I don't really know where to even start this at >.< any help is greatly appreciated.
 
Physics news on Phys.org
Write grad(f) and grad(g) out in components and take their cross product. Now take div of that. Use that mixed partial derivatives are equal e.g. d^2(f)/(dxdy)=d^2(f)/(dydx). That will get the job done.
 
Personally, I like using index notation for this type of problem; it's a good deal more compact and, once you get used to it, very transparent. Instead of writing out all the components longhand, write \nabla f \times \nabla g = \tilde{\varepsilon}^{ijk} f_{,i} g_{,j} \hat{\mathbf{e}}_{k} , where \tilde{\varepsilon} is the Levi-Civita symbol (not being careful about index placement here, since we're in flat space). Then you have
<br /> \begin{align*}<br /> \nabla \cdot (\nabla f \times \nabla g) &amp;= \tilde{\varepsilon}^{ijk} (f_{,i} g_{,j})_{,k}\\<br /> &amp;= \tilde{\varepsilon}^{ijk} f_{,i,k} g_{,j} + \tilde{\varepsilon}^{ijk} f_{,i} g_{,j,k} \textrm{.}<br /> \end{align*}<br />
Both summands in the last equation vanish. Indeed, let P = \tilde{\varepsilon}^{ijk} f_{,i,k} g_{,j} . Then, since partials commute, we have P = \tilde{\varepsilon}^{ijk} f_{,k,i} g_{,j} ; by a trivial relabeling of dummy indices, this gives P = \tilde{\varepsilon}^{kji} f_{,i,k} g_{,j} = -\tilde{\varepsilon}^{ijk} f_{,i,k} g_{,j} = -P (since \tilde{\varepsilon} is completely antisymmetric), so P = 0. The logic for the other summand is identical.
 
Alternatively, if you've already proven the following identities in class, you can simply combine them with \textbf{A}=\mathbf{\nabla}f and \textbf{B}=\mathbf{\nabla}g and the result becomes apparent.

Identities:

\mathbf{\nabla}\cdot(\textbf{A}\times\textbf{B})=\textbf{B}\cdot(\mathbf{\nabla}\times\textbf{A})-\textbf{B}\cdot(\mathbf{\nabla}\times\textbf{B})

\mathbf{\nabla}\times(\mthbf{\nabla}f)=0
 
Question: A clock's minute hand has length 4 and its hour hand has length 3. What is the distance between the tips at the moment when it is increasing most rapidly?(Putnam Exam Question) Answer: Making assumption that both the hands moves at constant angular velocities, the answer is ## \sqrt{7} .## But don't you think this assumption is somewhat doubtful and wrong?

Similar threads

  • · Replies 11 ·
Replies
11
Views
4K
  • · Replies 8 ·
Replies
8
Views
2K
  • · Replies 22 ·
Replies
22
Views
4K
  • · Replies 4 ·
Replies
4
Views
2K
Replies
1
Views
2K
  • · Replies 13 ·
Replies
13
Views
4K
Replies
4
Views
2K
Replies
4
Views
2K
  • · Replies 5 ·
Replies
5
Views
1K
  • · Replies 9 ·
Replies
9
Views
2K